How to get a smooth, uniform ParametricPlot of a 2D Region?How to plot a complicated Region?How to exclude a region from ParametricPlotHow discretize a region placing vertices on a specific non-uniform gridHow to transform a Plot or a ParametricPlot into a RegionHow can I get a smooth plot of a bounded region?Smooth ParametricPlot3D with RegionFunction?Smooth border of a region ParametricPlotSmooth region boundarySmooth region plot from list of pointsGet minimum y of a certain x in a region

Did "2001: A Space Odyssey" make any reference to the names of companies, or show any evidence of the existence of advertisements?

Pros and cons of playing correspondence chess

How do professors and lecturers learn to teach?

Is it OK for a Buddhist teacher to charge their students an hourly rate for their time?

Do insurance rates depend on credit scores?

Why does California seem to have much more aggressive Consumer Protection and Safety Legislation?

Query more than 50000 records

Is this medieval picture of hanging 5 royals showing an historical event?

Film about the USA being run by fake videos of the president after his kidnapping

Is American Express widely accepted in Hong Kong?

How many atoms in the hydrocarbon?

How do you give spell casters xp?

If a picture of a screen is a screenshot, what is a video of a screen?

Is there any plausible in-between of Endotherms and Ectotherms?

How can we save ourselves from large drops in stock price?

How would you separate fields with multiple spaces and store them in an array?

Reconstructed PIE grammar? Could we be able to speak in Proto-European?

Why do we have to discharge the capacitor before testing it in an LCR Meter?

Group Delay for Hilbert Transformer and Resulting Dispersion

Is there a material or method to allow "swimmable" coins?

Can any number of squares be a square?

Is there a BIOS setting that controls cpu load sharing?

Can the category of partial orders be fully embedded in the category of linear orders?

C++ Leak detection simple class



How to get a smooth, uniform ParametricPlot of a 2D Region?


How to plot a complicated Region?How to exclude a region from ParametricPlotHow discretize a region placing vertices on a specific non-uniform gridHow to transform a Plot or a ParametricPlot into a RegionHow can I get a smooth plot of a bounded region?Smooth ParametricPlot3D with RegionFunction?Smooth border of a region ParametricPlotSmooth region boundarySmooth region plot from list of pointsGet minimum y of a certain x in a region






.everyoneloves__top-leaderboard:empty,.everyoneloves__mid-leaderboard:empty,.everyoneloves__bot-mid-leaderboard:empty
margin-bottom:0;

.everyonelovesstackoverflowposition:absolute;height:1px;width:1px;opacity:0;top:0;left:0;pointer-events:none;








3















$begingroup$


This is an offspring of this question.




How to get a nice, smooth, uniform plot of the following? I.e. with no horizontal lines, and no ragged boundary at the top. I went with PlotPoints up to 400 and I'm dissapointed. What I'm actually after is a nicely Exported .pdf.



ParametricPlot[1 + a1/(-1 + a2), 
1 - 2 ArcCsc[2/Sqrt[1 + a1 - a2]]/π,
a2 >= -1 && 1 + a1 >= a2 && a1 + a2 <= 1, a1, -2, 2, a2, -1,
1, Frame -> True, PlotRange -> 0, 2, 0, 1,
AspectRatio -> 1/GoldenRatio, PlotPoints -> 150] // Quiet


enter image description here










share|improve this question









$endgroup$










  • 1




    $begingroup$
    ParametricPlot doesn't allow constraint to be passed in that way.
    $endgroup$
    – Chip Hurst
    Sep 28 at 13:21

















3















$begingroup$


This is an offspring of this question.




How to get a nice, smooth, uniform plot of the following? I.e. with no horizontal lines, and no ragged boundary at the top. I went with PlotPoints up to 400 and I'm dissapointed. What I'm actually after is a nicely Exported .pdf.



ParametricPlot[1 + a1/(-1 + a2), 
1 - 2 ArcCsc[2/Sqrt[1 + a1 - a2]]/π,
a2 >= -1 && 1 + a1 >= a2 && a1 + a2 <= 1, a1, -2, 2, a2, -1,
1, Frame -> True, PlotRange -> 0, 2, 0, 1,
AspectRatio -> 1/GoldenRatio, PlotPoints -> 150] // Quiet


enter image description here










share|improve this question









$endgroup$










  • 1




    $begingroup$
    ParametricPlot doesn't allow constraint to be passed in that way.
    $endgroup$
    – Chip Hurst
    Sep 28 at 13:21













3













3









3





$begingroup$


This is an offspring of this question.




How to get a nice, smooth, uniform plot of the following? I.e. with no horizontal lines, and no ragged boundary at the top. I went with PlotPoints up to 400 and I'm dissapointed. What I'm actually after is a nicely Exported .pdf.



ParametricPlot[1 + a1/(-1 + a2), 
1 - 2 ArcCsc[2/Sqrt[1 + a1 - a2]]/π,
a2 >= -1 && 1 + a1 >= a2 && a1 + a2 <= 1, a1, -2, 2, a2, -1,
1, Frame -> True, PlotRange -> 0, 2, 0, 1,
AspectRatio -> 1/GoldenRatio, PlotPoints -> 150] // Quiet


enter image description here










share|improve this question









$endgroup$




This is an offspring of this question.




How to get a nice, smooth, uniform plot of the following? I.e. with no horizontal lines, and no ragged boundary at the top. I went with PlotPoints up to 400 and I'm dissapointed. What I'm actually after is a nicely Exported .pdf.



ParametricPlot[1 + a1/(-1 + a2), 
1 - 2 ArcCsc[2/Sqrt[1 + a1 - a2]]/π,
a2 >= -1 && 1 + a1 >= a2 && a1 + a2 <= 1, a1, -2, 2, a2, -1,
1, Frame -> True, PlotRange -> 0, 2, 0, 1,
AspectRatio -> 1/GoldenRatio, PlotPoints -> 150] // Quiet


enter image description here







plotting regions






share|improve this question













share|improve this question











share|improve this question




share|improve this question










asked Sep 28 at 10:53









corey979corey979

21.9k6 gold badges44 silver badges83 bronze badges




21.9k6 gold badges44 silver badges83 bronze badges










  • 1




    $begingroup$
    ParametricPlot doesn't allow constraint to be passed in that way.
    $endgroup$
    – Chip Hurst
    Sep 28 at 13:21












  • 1




    $begingroup$
    ParametricPlot doesn't allow constraint to be passed in that way.
    $endgroup$
    – Chip Hurst
    Sep 28 at 13:21







1




1




$begingroup$
ParametricPlot doesn't allow constraint to be passed in that way.
$endgroup$
– Chip Hurst
Sep 28 at 13:21




$begingroup$
ParametricPlot doesn't allow constraint to be passed in that way.
$endgroup$
– Chip Hurst
Sep 28 at 13:21










3 Answers
3






active

oldest

votes


















2

















$begingroup$

To avoid the artifacts from singularities and jumps, we can take a somewhat manual approach.



Notice that the bottom boundary is formed from a2 == -1, the top boundary is a horizontal line formed as a2 -> 1 from the left, and the left boundary is a vertical line formed as a2 sweeps from -1 to 1.



So we can get a clean graphic by plotting the bottom boundary by fixing a2 == -1, extracting the points, and adding the upper left corner to form a polygon.



bdplot = With[a2 = -1,
ParametricPlot[1 + a1/(-1 + a2), 1 - 2 ArcCsc[2/Sqrt[1 + a1 - a2]]/π, a1, -2, 2,
Frame -> True, PlotRange -> 0, 2, 0, 1, AspectRatio -> 1/GoldenRatio] // Quiet
]




pts = Append[MeshCoordinates[DiscretizeGraphics[bdplot]], 0, 1];

poly = Polygon[FindShortestTour[pts][[2, 1 ;; -2]]];

Graphics[GraphicsComplex[pts, EdgeForm[], Hue[0.6, 0.3, 0.95], poly], Frame -> True, AspectRatio -> 1/GoldenRatio]





Now notice that your constraint is not needed:



Reduce[a2 >= -1 && 1 + a1 >= a2 && a1 + a2 <= 1, a1]



(-1 <= a2 < 1 && -1 + a2 <= a1 <= 1 - a2) || (a2 == 1 && a1 == 0)



We see the constraint says a1 should range from -1 + a2 to 1 - a2 instead of -2 to 2. If we plot for many fixed values of a2, we see we'd have the same plot if all a2 were sampled:



Show@Table[
ParametricPlot[1 + a1/(-1 + a2), 1 - 2 ArcCsc[2/Sqrt[1 + a1 - a2]]/π, a1, -1 + a2, 1 - a2,
Frame -> True, PlotRange -> 0, 2, 0, 1, AspectRatio -> 1/GoldenRatio, PlotPoints -> 100] // Quiet,
a2, Range[-1, 1, .01] /. -1. -> -0.999, 1. -> 0.999
]







share|improve this answer












$endgroup$













  • $begingroup$
    Exactly the approach I've just undertaken :o Although I've made a few tweaks. Will post in a moment.
    $endgroup$
    – corey979
    Sep 28 at 14:12










  • $begingroup$
    The lower boundray isn't th problem I think. The upper boundary, especially the point a1==2&&a2==1 has to be examined in more detail.
    $endgroup$
    – Ulrich Neumann
    Sep 29 at 5:57


















2

















$begingroup$

As discussed by Chip Hurst, the lower boundary of the region can be obtained by setting a2=-1. Therefore, this boundary is parametrized by a1 only (let it be called $(A,T)$):



reg = With[a2 = -1, 1 + a1/(-1 + a2), 1 - 2 ArcCsc[2/Sqrt[1 + a1 - a2]]/[Pi]]



1 - a1/2, 1 - (2 ArcCsc[2/Sqrt[2 + a1]])/[Pi]




This can be solved to get a1 as a function of A:



sol = Solve[A == reg[[1]], a1][[1]]



a1 -> -2 (-1 + A)




and inserted into T to obtain a function $T(A)$, Then the plotting is done with Filling:



Plot[reg[[2]] /. sol, A, 0, 2, Frame -> True, Filling -> Top, PlotStyle -> None]


enter image description here



The region can then be describe with e.g. ImplicitRegion.






share|improve this answer












$endgroup$





















    1

















    $begingroup$

    Try option RegionFunction inside ParametricPlot together with the Option MaxRecursions.



    The second plot argument 1 - 2 ArcCsc[2/Sqrt[1 + a1 - a2]]/[Pi] is only defined for 1 + a1 >= a2, that's why I only consider this restriction!



    ParametricPlot[ 1 + a1/(-1 + a2) ,1 - 2 ArcCsc[2/Sqrt[1 + a1 - a2]]/[Pi] , a1, -2, 2, a2, -1, 1,Frame -> True, PlotRange -> 0, 2 , 0, 1 ,AspectRatio -> 1/GoldenRatio, Evaluated -> True, MaxRecursion -> 4,PlotPoints->50, FrameLabel -> a1, a2,RegionFunction -> Function[x,y,a1, a2, -a1 + a2 <= 1 ]]


    enter image description here






    share|improve this answer












    $endgroup$













    • $begingroup$
      With a1, -2, 2, a2, -1, 1 and PlotRange -> 0, 2, 0, 1, the proper ranges I'm interested in, this plots a different Region than appears in my OP. And fyi, it's not (a1,a2) on the axes, but some functions of them.
      $endgroup$
      – corey979
      Sep 28 at 12:37











    • $begingroup$
      The scaling of the plotrange shouldn't be a problem I think. I modified my answer!
      $endgroup$
      – Ulrich Neumann
      Sep 28 at 12:42










    • $begingroup$
      I think your RegionFunction arg spec should be Function[x, y, a1, a2, ...].
      $endgroup$
      – Chip Hurst
      Sep 28 at 13:20











    • $begingroup$
      @ChipHurst Thanks, I changed my answer!
      $endgroup$
      – Ulrich Neumann
      Sep 29 at 5:54












    Your Answer








    StackExchange.ready(function()
    var channelOptions =
    tags: "".split(" "),
    id: "387"
    ;
    initTagRenderer("".split(" "), "".split(" "), channelOptions);

    StackExchange.using("externalEditor", function()
    // Have to fire editor after snippets, if snippets enabled
    if (StackExchange.settings.snippets.snippetsEnabled)
    StackExchange.using("snippets", function()
    createEditor();
    );

    else
    createEditor();

    );

    function createEditor()
    StackExchange.prepareEditor(
    heartbeatType: 'answer',
    autoActivateHeartbeat: false,
    convertImagesToLinks: false,
    noModals: true,
    showLowRepImageUploadWarning: true,
    reputationToPostImages: null,
    bindNavPrevention: true,
    postfix: "",
    imageUploader:
    brandingHtml: "Powered by u003ca class="icon-imgur-white" href="https://imgur.com/"u003eu003c/au003e",
    contentPolicyHtml: "User contributions licensed under u003ca href="https://creativecommons.org/licenses/by-sa/4.0/"u003ecc by-sa 4.0 with attribution requiredu003c/au003e u003ca href="https://stackoverflow.com/legal/content-policy"u003e(content policy)u003c/au003e",
    allowUrls: true
    ,
    onDemand: true,
    discardSelector: ".discard-answer"
    ,immediatelyShowMarkdownHelp:true
    );



    );














    draft saved

    draft discarded
















    StackExchange.ready(
    function ()
    StackExchange.openid.initPostLogin('.new-post-login', 'https%3a%2f%2fmathematica.stackexchange.com%2fquestions%2f207010%2fhow-to-get-a-smooth-uniform-parametricplot-of-a-2d-region%23new-answer', 'question_page');

    );

    Post as a guest















    Required, but never shown


























    3 Answers
    3






    active

    oldest

    votes








    3 Answers
    3






    active

    oldest

    votes









    active

    oldest

    votes






    active

    oldest

    votes









    2

















    $begingroup$

    To avoid the artifacts from singularities and jumps, we can take a somewhat manual approach.



    Notice that the bottom boundary is formed from a2 == -1, the top boundary is a horizontal line formed as a2 -> 1 from the left, and the left boundary is a vertical line formed as a2 sweeps from -1 to 1.



    So we can get a clean graphic by plotting the bottom boundary by fixing a2 == -1, extracting the points, and adding the upper left corner to form a polygon.



    bdplot = With[a2 = -1,
    ParametricPlot[1 + a1/(-1 + a2), 1 - 2 ArcCsc[2/Sqrt[1 + a1 - a2]]/π, a1, -2, 2,
    Frame -> True, PlotRange -> 0, 2, 0, 1, AspectRatio -> 1/GoldenRatio] // Quiet
    ]




    pts = Append[MeshCoordinates[DiscretizeGraphics[bdplot]], 0, 1];

    poly = Polygon[FindShortestTour[pts][[2, 1 ;; -2]]];

    Graphics[GraphicsComplex[pts, EdgeForm[], Hue[0.6, 0.3, 0.95], poly], Frame -> True, AspectRatio -> 1/GoldenRatio]





    Now notice that your constraint is not needed:



    Reduce[a2 >= -1 && 1 + a1 >= a2 && a1 + a2 <= 1, a1]



    (-1 <= a2 < 1 && -1 + a2 <= a1 <= 1 - a2) || (a2 == 1 && a1 == 0)



    We see the constraint says a1 should range from -1 + a2 to 1 - a2 instead of -2 to 2. If we plot for many fixed values of a2, we see we'd have the same plot if all a2 were sampled:



    Show@Table[
    ParametricPlot[1 + a1/(-1 + a2), 1 - 2 ArcCsc[2/Sqrt[1 + a1 - a2]]/π, a1, -1 + a2, 1 - a2,
    Frame -> True, PlotRange -> 0, 2, 0, 1, AspectRatio -> 1/GoldenRatio, PlotPoints -> 100] // Quiet,
    a2, Range[-1, 1, .01] /. -1. -> -0.999, 1. -> 0.999
    ]







    share|improve this answer












    $endgroup$













    • $begingroup$
      Exactly the approach I've just undertaken :o Although I've made a few tweaks. Will post in a moment.
      $endgroup$
      – corey979
      Sep 28 at 14:12










    • $begingroup$
      The lower boundray isn't th problem I think. The upper boundary, especially the point a1==2&&a2==1 has to be examined in more detail.
      $endgroup$
      – Ulrich Neumann
      Sep 29 at 5:57















    2

















    $begingroup$

    To avoid the artifacts from singularities and jumps, we can take a somewhat manual approach.



    Notice that the bottom boundary is formed from a2 == -1, the top boundary is a horizontal line formed as a2 -> 1 from the left, and the left boundary is a vertical line formed as a2 sweeps from -1 to 1.



    So we can get a clean graphic by plotting the bottom boundary by fixing a2 == -1, extracting the points, and adding the upper left corner to form a polygon.



    bdplot = With[a2 = -1,
    ParametricPlot[1 + a1/(-1 + a2), 1 - 2 ArcCsc[2/Sqrt[1 + a1 - a2]]/π, a1, -2, 2,
    Frame -> True, PlotRange -> 0, 2, 0, 1, AspectRatio -> 1/GoldenRatio] // Quiet
    ]




    pts = Append[MeshCoordinates[DiscretizeGraphics[bdplot]], 0, 1];

    poly = Polygon[FindShortestTour[pts][[2, 1 ;; -2]]];

    Graphics[GraphicsComplex[pts, EdgeForm[], Hue[0.6, 0.3, 0.95], poly], Frame -> True, AspectRatio -> 1/GoldenRatio]





    Now notice that your constraint is not needed:



    Reduce[a2 >= -1 && 1 + a1 >= a2 && a1 + a2 <= 1, a1]



    (-1 <= a2 < 1 && -1 + a2 <= a1 <= 1 - a2) || (a2 == 1 && a1 == 0)



    We see the constraint says a1 should range from -1 + a2 to 1 - a2 instead of -2 to 2. If we plot for many fixed values of a2, we see we'd have the same plot if all a2 were sampled:



    Show@Table[
    ParametricPlot[1 + a1/(-1 + a2), 1 - 2 ArcCsc[2/Sqrt[1 + a1 - a2]]/π, a1, -1 + a2, 1 - a2,
    Frame -> True, PlotRange -> 0, 2, 0, 1, AspectRatio -> 1/GoldenRatio, PlotPoints -> 100] // Quiet,
    a2, Range[-1, 1, .01] /. -1. -> -0.999, 1. -> 0.999
    ]







    share|improve this answer












    $endgroup$













    • $begingroup$
      Exactly the approach I've just undertaken :o Although I've made a few tweaks. Will post in a moment.
      $endgroup$
      – corey979
      Sep 28 at 14:12










    • $begingroup$
      The lower boundray isn't th problem I think. The upper boundary, especially the point a1==2&&a2==1 has to be examined in more detail.
      $endgroup$
      – Ulrich Neumann
      Sep 29 at 5:57













    2















    2











    2







    $begingroup$

    To avoid the artifacts from singularities and jumps, we can take a somewhat manual approach.



    Notice that the bottom boundary is formed from a2 == -1, the top boundary is a horizontal line formed as a2 -> 1 from the left, and the left boundary is a vertical line formed as a2 sweeps from -1 to 1.



    So we can get a clean graphic by plotting the bottom boundary by fixing a2 == -1, extracting the points, and adding the upper left corner to form a polygon.



    bdplot = With[a2 = -1,
    ParametricPlot[1 + a1/(-1 + a2), 1 - 2 ArcCsc[2/Sqrt[1 + a1 - a2]]/π, a1, -2, 2,
    Frame -> True, PlotRange -> 0, 2, 0, 1, AspectRatio -> 1/GoldenRatio] // Quiet
    ]




    pts = Append[MeshCoordinates[DiscretizeGraphics[bdplot]], 0, 1];

    poly = Polygon[FindShortestTour[pts][[2, 1 ;; -2]]];

    Graphics[GraphicsComplex[pts, EdgeForm[], Hue[0.6, 0.3, 0.95], poly], Frame -> True, AspectRatio -> 1/GoldenRatio]





    Now notice that your constraint is not needed:



    Reduce[a2 >= -1 && 1 + a1 >= a2 && a1 + a2 <= 1, a1]



    (-1 <= a2 < 1 && -1 + a2 <= a1 <= 1 - a2) || (a2 == 1 && a1 == 0)



    We see the constraint says a1 should range from -1 + a2 to 1 - a2 instead of -2 to 2. If we plot for many fixed values of a2, we see we'd have the same plot if all a2 were sampled:



    Show@Table[
    ParametricPlot[1 + a1/(-1 + a2), 1 - 2 ArcCsc[2/Sqrt[1 + a1 - a2]]/π, a1, -1 + a2, 1 - a2,
    Frame -> True, PlotRange -> 0, 2, 0, 1, AspectRatio -> 1/GoldenRatio, PlotPoints -> 100] // Quiet,
    a2, Range[-1, 1, .01] /. -1. -> -0.999, 1. -> 0.999
    ]







    share|improve this answer












    $endgroup$



    To avoid the artifacts from singularities and jumps, we can take a somewhat manual approach.



    Notice that the bottom boundary is formed from a2 == -1, the top boundary is a horizontal line formed as a2 -> 1 from the left, and the left boundary is a vertical line formed as a2 sweeps from -1 to 1.



    So we can get a clean graphic by plotting the bottom boundary by fixing a2 == -1, extracting the points, and adding the upper left corner to form a polygon.



    bdplot = With[a2 = -1,
    ParametricPlot[1 + a1/(-1 + a2), 1 - 2 ArcCsc[2/Sqrt[1 + a1 - a2]]/π, a1, -2, 2,
    Frame -> True, PlotRange -> 0, 2, 0, 1, AspectRatio -> 1/GoldenRatio] // Quiet
    ]




    pts = Append[MeshCoordinates[DiscretizeGraphics[bdplot]], 0, 1];

    poly = Polygon[FindShortestTour[pts][[2, 1 ;; -2]]];

    Graphics[GraphicsComplex[pts, EdgeForm[], Hue[0.6, 0.3, 0.95], poly], Frame -> True, AspectRatio -> 1/GoldenRatio]





    Now notice that your constraint is not needed:



    Reduce[a2 >= -1 && 1 + a1 >= a2 && a1 + a2 <= 1, a1]



    (-1 <= a2 < 1 && -1 + a2 <= a1 <= 1 - a2) || (a2 == 1 && a1 == 0)



    We see the constraint says a1 should range from -1 + a2 to 1 - a2 instead of -2 to 2. If we plot for many fixed values of a2, we see we'd have the same plot if all a2 were sampled:



    Show@Table[
    ParametricPlot[1 + a1/(-1 + a2), 1 - 2 ArcCsc[2/Sqrt[1 + a1 - a2]]/π, a1, -1 + a2, 1 - a2,
    Frame -> True, PlotRange -> 0, 2, 0, 1, AspectRatio -> 1/GoldenRatio, PlotPoints -> 100] // Quiet,
    a2, Range[-1, 1, .01] /. -1. -> -0.999, 1. -> 0.999
    ]








    share|improve this answer















    share|improve this answer




    share|improve this answer








    edited Sep 28 at 14:11

























    answered Sep 28 at 14:04









    Chip HurstChip Hurst

    26.4k1 gold badge65 silver badges105 bronze badges




    26.4k1 gold badge65 silver badges105 bronze badges














    • $begingroup$
      Exactly the approach I've just undertaken :o Although I've made a few tweaks. Will post in a moment.
      $endgroup$
      – corey979
      Sep 28 at 14:12










    • $begingroup$
      The lower boundray isn't th problem I think. The upper boundary, especially the point a1==2&&a2==1 has to be examined in more detail.
      $endgroup$
      – Ulrich Neumann
      Sep 29 at 5:57
















    • $begingroup$
      Exactly the approach I've just undertaken :o Although I've made a few tweaks. Will post in a moment.
      $endgroup$
      – corey979
      Sep 28 at 14:12










    • $begingroup$
      The lower boundray isn't th problem I think. The upper boundary, especially the point a1==2&&a2==1 has to be examined in more detail.
      $endgroup$
      – Ulrich Neumann
      Sep 29 at 5:57















    $begingroup$
    Exactly the approach I've just undertaken :o Although I've made a few tweaks. Will post in a moment.
    $endgroup$
    – corey979
    Sep 28 at 14:12




    $begingroup$
    Exactly the approach I've just undertaken :o Although I've made a few tweaks. Will post in a moment.
    $endgroup$
    – corey979
    Sep 28 at 14:12












    $begingroup$
    The lower boundray isn't th problem I think. The upper boundary, especially the point a1==2&&a2==1 has to be examined in more detail.
    $endgroup$
    – Ulrich Neumann
    Sep 29 at 5:57




    $begingroup$
    The lower boundray isn't th problem I think. The upper boundary, especially the point a1==2&&a2==1 has to be examined in more detail.
    $endgroup$
    – Ulrich Neumann
    Sep 29 at 5:57













    2

















    $begingroup$

    As discussed by Chip Hurst, the lower boundary of the region can be obtained by setting a2=-1. Therefore, this boundary is parametrized by a1 only (let it be called $(A,T)$):



    reg = With[a2 = -1, 1 + a1/(-1 + a2), 1 - 2 ArcCsc[2/Sqrt[1 + a1 - a2]]/[Pi]]



    1 - a1/2, 1 - (2 ArcCsc[2/Sqrt[2 + a1]])/[Pi]




    This can be solved to get a1 as a function of A:



    sol = Solve[A == reg[[1]], a1][[1]]



    a1 -> -2 (-1 + A)




    and inserted into T to obtain a function $T(A)$, Then the plotting is done with Filling:



    Plot[reg[[2]] /. sol, A, 0, 2, Frame -> True, Filling -> Top, PlotStyle -> None]


    enter image description here



    The region can then be describe with e.g. ImplicitRegion.






    share|improve this answer












    $endgroup$


















      2

















      $begingroup$

      As discussed by Chip Hurst, the lower boundary of the region can be obtained by setting a2=-1. Therefore, this boundary is parametrized by a1 only (let it be called $(A,T)$):



      reg = With[a2 = -1, 1 + a1/(-1 + a2), 1 - 2 ArcCsc[2/Sqrt[1 + a1 - a2]]/[Pi]]



      1 - a1/2, 1 - (2 ArcCsc[2/Sqrt[2 + a1]])/[Pi]




      This can be solved to get a1 as a function of A:



      sol = Solve[A == reg[[1]], a1][[1]]



      a1 -> -2 (-1 + A)




      and inserted into T to obtain a function $T(A)$, Then the plotting is done with Filling:



      Plot[reg[[2]] /. sol, A, 0, 2, Frame -> True, Filling -> Top, PlotStyle -> None]


      enter image description here



      The region can then be describe with e.g. ImplicitRegion.






      share|improve this answer












      $endgroup$
















        2















        2











        2







        $begingroup$

        As discussed by Chip Hurst, the lower boundary of the region can be obtained by setting a2=-1. Therefore, this boundary is parametrized by a1 only (let it be called $(A,T)$):



        reg = With[a2 = -1, 1 + a1/(-1 + a2), 1 - 2 ArcCsc[2/Sqrt[1 + a1 - a2]]/[Pi]]



        1 - a1/2, 1 - (2 ArcCsc[2/Sqrt[2 + a1]])/[Pi]




        This can be solved to get a1 as a function of A:



        sol = Solve[A == reg[[1]], a1][[1]]



        a1 -> -2 (-1 + A)




        and inserted into T to obtain a function $T(A)$, Then the plotting is done with Filling:



        Plot[reg[[2]] /. sol, A, 0, 2, Frame -> True, Filling -> Top, PlotStyle -> None]


        enter image description here



        The region can then be describe with e.g. ImplicitRegion.






        share|improve this answer












        $endgroup$



        As discussed by Chip Hurst, the lower boundary of the region can be obtained by setting a2=-1. Therefore, this boundary is parametrized by a1 only (let it be called $(A,T)$):



        reg = With[a2 = -1, 1 + a1/(-1 + a2), 1 - 2 ArcCsc[2/Sqrt[1 + a1 - a2]]/[Pi]]



        1 - a1/2, 1 - (2 ArcCsc[2/Sqrt[2 + a1]])/[Pi]




        This can be solved to get a1 as a function of A:



        sol = Solve[A == reg[[1]], a1][[1]]



        a1 -> -2 (-1 + A)




        and inserted into T to obtain a function $T(A)$, Then the plotting is done with Filling:



        Plot[reg[[2]] /. sol, A, 0, 2, Frame -> True, Filling -> Top, PlotStyle -> None]


        enter image description here



        The region can then be describe with e.g. ImplicitRegion.







        share|improve this answer















        share|improve this answer




        share|improve this answer








        edited Sep 28 at 14:51

























        answered Sep 28 at 14:19









        corey979corey979

        21.9k6 gold badges44 silver badges83 bronze badges




        21.9k6 gold badges44 silver badges83 bronze badges
























            1

















            $begingroup$

            Try option RegionFunction inside ParametricPlot together with the Option MaxRecursions.



            The second plot argument 1 - 2 ArcCsc[2/Sqrt[1 + a1 - a2]]/[Pi] is only defined for 1 + a1 >= a2, that's why I only consider this restriction!



            ParametricPlot[ 1 + a1/(-1 + a2) ,1 - 2 ArcCsc[2/Sqrt[1 + a1 - a2]]/[Pi] , a1, -2, 2, a2, -1, 1,Frame -> True, PlotRange -> 0, 2 , 0, 1 ,AspectRatio -> 1/GoldenRatio, Evaluated -> True, MaxRecursion -> 4,PlotPoints->50, FrameLabel -> a1, a2,RegionFunction -> Function[x,y,a1, a2, -a1 + a2 <= 1 ]]


            enter image description here






            share|improve this answer












            $endgroup$













            • $begingroup$
              With a1, -2, 2, a2, -1, 1 and PlotRange -> 0, 2, 0, 1, the proper ranges I'm interested in, this plots a different Region than appears in my OP. And fyi, it's not (a1,a2) on the axes, but some functions of them.
              $endgroup$
              – corey979
              Sep 28 at 12:37











            • $begingroup$
              The scaling of the plotrange shouldn't be a problem I think. I modified my answer!
              $endgroup$
              – Ulrich Neumann
              Sep 28 at 12:42










            • $begingroup$
              I think your RegionFunction arg spec should be Function[x, y, a1, a2, ...].
              $endgroup$
              – Chip Hurst
              Sep 28 at 13:20











            • $begingroup$
              @ChipHurst Thanks, I changed my answer!
              $endgroup$
              – Ulrich Neumann
              Sep 29 at 5:54















            1

















            $begingroup$

            Try option RegionFunction inside ParametricPlot together with the Option MaxRecursions.



            The second plot argument 1 - 2 ArcCsc[2/Sqrt[1 + a1 - a2]]/[Pi] is only defined for 1 + a1 >= a2, that's why I only consider this restriction!



            ParametricPlot[ 1 + a1/(-1 + a2) ,1 - 2 ArcCsc[2/Sqrt[1 + a1 - a2]]/[Pi] , a1, -2, 2, a2, -1, 1,Frame -> True, PlotRange -> 0, 2 , 0, 1 ,AspectRatio -> 1/GoldenRatio, Evaluated -> True, MaxRecursion -> 4,PlotPoints->50, FrameLabel -> a1, a2,RegionFunction -> Function[x,y,a1, a2, -a1 + a2 <= 1 ]]


            enter image description here






            share|improve this answer












            $endgroup$













            • $begingroup$
              With a1, -2, 2, a2, -1, 1 and PlotRange -> 0, 2, 0, 1, the proper ranges I'm interested in, this plots a different Region than appears in my OP. And fyi, it's not (a1,a2) on the axes, but some functions of them.
              $endgroup$
              – corey979
              Sep 28 at 12:37











            • $begingroup$
              The scaling of the plotrange shouldn't be a problem I think. I modified my answer!
              $endgroup$
              – Ulrich Neumann
              Sep 28 at 12:42










            • $begingroup$
              I think your RegionFunction arg spec should be Function[x, y, a1, a2, ...].
              $endgroup$
              – Chip Hurst
              Sep 28 at 13:20











            • $begingroup$
              @ChipHurst Thanks, I changed my answer!
              $endgroup$
              – Ulrich Neumann
              Sep 29 at 5:54













            1















            1











            1







            $begingroup$

            Try option RegionFunction inside ParametricPlot together with the Option MaxRecursions.



            The second plot argument 1 - 2 ArcCsc[2/Sqrt[1 + a1 - a2]]/[Pi] is only defined for 1 + a1 >= a2, that's why I only consider this restriction!



            ParametricPlot[ 1 + a1/(-1 + a2) ,1 - 2 ArcCsc[2/Sqrt[1 + a1 - a2]]/[Pi] , a1, -2, 2, a2, -1, 1,Frame -> True, PlotRange -> 0, 2 , 0, 1 ,AspectRatio -> 1/GoldenRatio, Evaluated -> True, MaxRecursion -> 4,PlotPoints->50, FrameLabel -> a1, a2,RegionFunction -> Function[x,y,a1, a2, -a1 + a2 <= 1 ]]


            enter image description here






            share|improve this answer












            $endgroup$



            Try option RegionFunction inside ParametricPlot together with the Option MaxRecursions.



            The second plot argument 1 - 2 ArcCsc[2/Sqrt[1 + a1 - a2]]/[Pi] is only defined for 1 + a1 >= a2, that's why I only consider this restriction!



            ParametricPlot[ 1 + a1/(-1 + a2) ,1 - 2 ArcCsc[2/Sqrt[1 + a1 - a2]]/[Pi] , a1, -2, 2, a2, -1, 1,Frame -> True, PlotRange -> 0, 2 , 0, 1 ,AspectRatio -> 1/GoldenRatio, Evaluated -> True, MaxRecursion -> 4,PlotPoints->50, FrameLabel -> a1, a2,RegionFunction -> Function[x,y,a1, a2, -a1 + a2 <= 1 ]]


            enter image description here







            share|improve this answer















            share|improve this answer




            share|improve this answer








            edited Sep 28 at 13:39

























            answered Sep 28 at 12:30









            Ulrich NeumannUlrich Neumann

            16.8k9 silver badges24 bronze badges




            16.8k9 silver badges24 bronze badges














            • $begingroup$
              With a1, -2, 2, a2, -1, 1 and PlotRange -> 0, 2, 0, 1, the proper ranges I'm interested in, this plots a different Region than appears in my OP. And fyi, it's not (a1,a2) on the axes, but some functions of them.
              $endgroup$
              – corey979
              Sep 28 at 12:37











            • $begingroup$
              The scaling of the plotrange shouldn't be a problem I think. I modified my answer!
              $endgroup$
              – Ulrich Neumann
              Sep 28 at 12:42










            • $begingroup$
              I think your RegionFunction arg spec should be Function[x, y, a1, a2, ...].
              $endgroup$
              – Chip Hurst
              Sep 28 at 13:20











            • $begingroup$
              @ChipHurst Thanks, I changed my answer!
              $endgroup$
              – Ulrich Neumann
              Sep 29 at 5:54
















            • $begingroup$
              With a1, -2, 2, a2, -1, 1 and PlotRange -> 0, 2, 0, 1, the proper ranges I'm interested in, this plots a different Region than appears in my OP. And fyi, it's not (a1,a2) on the axes, but some functions of them.
              $endgroup$
              – corey979
              Sep 28 at 12:37











            • $begingroup$
              The scaling of the plotrange shouldn't be a problem I think. I modified my answer!
              $endgroup$
              – Ulrich Neumann
              Sep 28 at 12:42










            • $begingroup$
              I think your RegionFunction arg spec should be Function[x, y, a1, a2, ...].
              $endgroup$
              – Chip Hurst
              Sep 28 at 13:20











            • $begingroup$
              @ChipHurst Thanks, I changed my answer!
              $endgroup$
              – Ulrich Neumann
              Sep 29 at 5:54















            $begingroup$
            With a1, -2, 2, a2, -1, 1 and PlotRange -> 0, 2, 0, 1, the proper ranges I'm interested in, this plots a different Region than appears in my OP. And fyi, it's not (a1,a2) on the axes, but some functions of them.
            $endgroup$
            – corey979
            Sep 28 at 12:37





            $begingroup$
            With a1, -2, 2, a2, -1, 1 and PlotRange -> 0, 2, 0, 1, the proper ranges I'm interested in, this plots a different Region than appears in my OP. And fyi, it's not (a1,a2) on the axes, but some functions of them.
            $endgroup$
            – corey979
            Sep 28 at 12:37













            $begingroup$
            The scaling of the plotrange shouldn't be a problem I think. I modified my answer!
            $endgroup$
            – Ulrich Neumann
            Sep 28 at 12:42




            $begingroup$
            The scaling of the plotrange shouldn't be a problem I think. I modified my answer!
            $endgroup$
            – Ulrich Neumann
            Sep 28 at 12:42












            $begingroup$
            I think your RegionFunction arg spec should be Function[x, y, a1, a2, ...].
            $endgroup$
            – Chip Hurst
            Sep 28 at 13:20





            $begingroup$
            I think your RegionFunction arg spec should be Function[x, y, a1, a2, ...].
            $endgroup$
            – Chip Hurst
            Sep 28 at 13:20













            $begingroup$
            @ChipHurst Thanks, I changed my answer!
            $endgroup$
            – Ulrich Neumann
            Sep 29 at 5:54




            $begingroup$
            @ChipHurst Thanks, I changed my answer!
            $endgroup$
            – Ulrich Neumann
            Sep 29 at 5:54


















            draft saved

            draft discarded















































            Thanks for contributing an answer to Mathematica Stack Exchange!


            • Please be sure to answer the question. Provide details and share your research!

            But avoid


            • Asking for help, clarification, or responding to other answers.

            • Making statements based on opinion; back them up with references or personal experience.

            Use MathJax to format equations. MathJax reference.


            To learn more, see our tips on writing great answers.




            draft saved


            draft discarded














            StackExchange.ready(
            function ()
            StackExchange.openid.initPostLogin('.new-post-login', 'https%3a%2f%2fmathematica.stackexchange.com%2fquestions%2f207010%2fhow-to-get-a-smooth-uniform-parametricplot-of-a-2d-region%23new-answer', 'question_page');

            );

            Post as a guest















            Required, but never shown





















































            Required, but never shown














            Required, but never shown












            Required, but never shown







            Required, but never shown

































            Required, but never shown














            Required, but never shown












            Required, but never shown







            Required, but never shown









            Popular posts from this blog

            Tamil (spriik) Luke uk diar | Nawigatjuun

            Align equal signs while including text over equalitiesAMS align: left aligned text/math plus multicolumn alignmentMultiple alignmentsAligning equations in multiple placesNumbering and aligning an equation with multiple columnsHow to align one equation with another multline equationUsing \ in environments inside the begintabularxNumber equations and preserving alignment of equal signsHow can I align equations to the left and to the right?Double equation alignment problem within align enviromentAligned within align: Why are they right-aligned?

            Where does the image of a data connector as a sharp metal spike originate from?Where does the concept of infected people turning into zombies only after death originate from?Where does the motif of a reanimated human head originate?Where did the notion that Dragons could speak originate?Where does the archetypal image of the 'Grey' alien come from?Where did the suffix '-Man' originate?Where does the notion of being injured or killed by an illusion originate?Where did the term “sophont” originate?Where does the trope of magic spells being driven by advanced technology originate from?Where did the term “the living impaired” originate?